Question

1. What is the minimum telescope aperture needed to resolve an object whose angular diameter is...

1. What is the minimum telescope aperture needed to resolve an object whose angular diameter is 0.38 arc seconds, observed at 540 nm wavelength? Note: 1arcsec=(1/3600)∘. **express in cm

2. Find the minimum angular separation resolvable with 625 nm laser light passing through a circular aperture of diameter 2.1 cm .

3. The interference pattern from two slits separated by 0.35 mm has bright fringes with angular spacing 0.073 ∘. **express in nm**

0 0
Add a comment Improve this question Transcribed image text
Answer #1

1.

Qmin = 1.22 l/D where Q is the minimum resolvable angel in radians, D = aperture diameter, and l = wavelength.


Convert arsec to radians ---> Qmin = 1.84224x10^-6 radians

Solve for D:

D = 1.22 l/Qmin = 1.22* (540x10^-9 m)/(1.84224x10^-6 rad) = 0.35760 m

2.

     θ = 1.220 λ / D                     

θ is the angular resolution (radians),
λ is the wavelength of light,
and D is the diameter of the lens' aperture

θ = 1.220 *625*10^-9 / 2.1*10^-2

θ = 3.630*10^-50

3.

θ = λ/d
Convert mm to nm 10^-3

λ = Sin( .073) * 0.35 * 10^-3

    = 233 nm

Add a comment
Know the answer?
Add Answer to:
1. What is the minimum telescope aperture needed to resolve an object whose angular diameter is...
Your Answer:

Post as a guest

Your Name:

What's your source?

Earn Coins

Coins can be redeemed for fabulous gifts.

Not the answer you're looking for? Ask your own homework help question. Our experts will answer your question WITHIN MINUTES for Free.
Similar Homework Help Questions
  • Find the minimum angular separation resolvable with 625nm laser light passing through a circular aperture of...

    Find the minimum angular separation resolvable with 625nm laser light passing through a circular aperture of diameter 2.2cm . Express your answer using two significant figures.

  • Problem 2 a. Using the telescope on-board his spaceship, Captain Kirk barely manages to resolve two...

    Problem 2 a. Using the telescope on-board his spaceship, Captain Kirk barely manages to resolve two distant galaxies that are 0.3 arc-seconds apart (1 arc-second 1/3600 Degree). If he is imaging at 550 nm wavelength, what is the effective aperture (diameter) of the objective lens of the telescope? Assume that there are no geometrical aberrations. (5 points) b. Consider the following imaging setup with a transverse magnification Iml3, If we are to resolve the image of the two fixed point...

  • Constants Part A The resolution of a telescope is ultimately limited by the diameter of its...

    Constants Part A The resolution of a telescope is ultimately limited by the diameter of its objective lens or mirror. A typical amateur astronomer's telescope may have a 7.0 in. -diameter mirror What is the minimum angular separation (in arc seconds) of two stars that can be resolved with a 7.0 in. scope? (Take ? to be at the center of the visible spectrum, about 550 nm, and note that 1 arc sec 1/3600 ) Express your answer using two...

  • A telescope with aperture diameter 1.0 m is used to observe a globular star cluster. What...

    A telescope with aperture diameter 1.0 m is used to observe a globular star cluster. What is the best resolution possible for this telescope? That is, what is the minimum angular separation needed to distinguish two objects? a. 0.732 radians b. 0.6 degrees c. 0.49 radians d. 36.9 degrees

  • Suppose your Newtonian reflect has an objective mirror 10 cm (4 inches) in diameter. a) What...

    Suppose your Newtonian reflect has an objective mirror 10 cm (4 inches) in diameter. a) What is the telescope's diffraction-limited angular resolution when used with yellow light of wavelength 600 nm? b) A binary star is a system of two stars orbiting each other. Suppose you are observing a binary star consisting of two yellow stars having an angular separation of 0.5 arc-seconds. Will your telescope be able to resolve the two stars? Explain your answer. c) Would you be...

  • 1) A laser beam is incident on two slits with separation d = 0.026 mm. A...

    1) A laser beam is incident on two slits with separation d = 0.026 mm. A screen is placed L = 3.8 m from the slits. The wavelength of the laser light is λ = 4250 Å. θ1 and θ2 are the angles to the first and second bright fringes above the center of the screen. a) Express sin(θ1) in terms of d and λ: sin(θ1) = ____________    b) Express sin(θ2) in terms of d and λ: sin(θ2) =...

  • The Hubble Space Telescope has a diameter of 2.4 m. 1) What is the angular limit...

    The Hubble Space Telescope has a diameter of 2.4 m. 1) What is the angular limit of resolution due to diffraction when a wavelength of 990 nm is viewed? (Express your answer to two significant figures.)

  • 1 (a) What is the minimum angular spread (in rad) of a 694 nm wavelength ruby...

    1 (a) What is the minimum angular spread (in rad) of a 694 nm wavelength ruby laser beam that is originally 1.28 mm in diameter? ___ rad (b) If this laser is aimed at a mountain cliff 14.5 km away, how big will the illuminated spot be? (Give the diameter of the spot in meters.) ____ m (c) How big a spot would be illuminated on the Moon, neglecting atmospheric effects? (This might be done to hit a corner reflector...

  • 1) A diffraction grating is w-3 cm wide and has N-1.2 x 104 slits. A monochromatic...

    1) A diffraction grating is w-3 cm wide and has N-1.2 x 104 slits. A monochromatic light source of wavelength λ-680 nm shines through the grating onto a screen. At what angle does the first-order line appear on the screen? 8-1 5.22 deg 8-1 5.78 deg θ-105.32 deg θ-59.54 deg 0 143.24 deg Submit 2) Another source of light is present and is just barely resolved from the 680 nm source in the second order. What is the difference in...

  • 1) A diffraction grating is w-3 cm wide and has N-1.2 x 104 slits. A monochromatic...

    1) A diffraction grating is w-3 cm wide and has N-1.2 x 104 slits. A monochromatic light source of wavelength λ-680 nm shines through the grating onto a screen. At what angle does the first-order line appear on the screen? 8-1 5.22 deg 8-1 5.78 deg θ-105.32 deg θ-59.54 deg 0 143.24 deg Submit 2) Another source of light is present and is just barely resolved from the 680 nm source in the second order. What is the difference in...

ADVERTISEMENT
Free Homework Help App
Download From Google Play
Scan Your Homework
to Get Instant Free Answers
Need Online Homework Help?
Ask a Question
Get Answers For Free
Most questions answered within 3 hours.
ADVERTISEMENT
ADVERTISEMENT
ADVERTISEMENT